Difference between revisions of "1964 AHSME Problems/Problem 3"
Talkinaway (talk | contribs) |
(Solution $1$=) |
||
Line 13: | Line 13: | ||
==Solution 1== | ==Solution 1== | ||
− | + | *We can solve this problem by elemetary modular arthmetic, | |
+ | <math>x</math> \equiv. <math>v</math> (<math>mod y</math>) <math>=></math> <math>x</math> + <math>2uy</math> \equiv. <math>v</math> (<math>mod y</math>). | ||
− | The problem asks to find the remainder of <math>x + 2uy< | + | % Solution by GEOMETRY-WIZARD<math> |
+ | ==Solution 2== | ||
+ | |||
+ | By the definition of quotient and remainder, problem states that </math>x = uy + v<math>. | ||
+ | |||
+ | The problem asks to find the remainder of </math>x + 2uy<math> when divided by </math>y<math>. Since </math>2uy<math> is divisible by </math>y<math>, adding it to </math>x<math> will not change the remainder. Therefore, the answer is </math>\boxed{\textbf{(D)}}<math>. | ||
− | ==Solution | + | ==Solution 3== |
− | If the statement is true for all values of <math>(x, y, u, v)< | + | If the statement is true for all values of </math>(x, y, u, v)<math>, then it must be true for a specific set of </math>(x, y, u, v)<math>. |
− | If you let <math>x=43< | + | If you let </math>x=43<math> and </math>y = 8<math>, then the quotient is </math>u = 5<math> and the remainder is </math>v = 3<math>. The problem asks what the remainder is when you divide </math>x + 2uy = 43 + 2 \cdot 5 \cdot 8 = 123<math> by </math>8<math>. In this case, the remainder is </math>3<math>. |
− | When you plug in <math>u=5< | + | When you plug in </math>u=5<math> and </math>v = 3<math> into the answer choices, they become </math>0, 5, 10, 3, 6<math>, respectively. Therefore, the answer is </math>\boxed{\textbf{(D)}}$. |
==See Also== | ==See Also== |
Revision as of 04:39, 31 December 2023
Problem
When a positive integer is divided by a positive integer , the quotient is and the remainder is , where and are integers. What is the remainder when is divided by ?
Solution 1
- We can solve this problem by elemetary modular arthmetic,
\equiv. () + \equiv. ().
% Solution by GEOMETRY-WIZARD$==Solution 2==
By the definition of quotient and remainder, problem states that$ (Error compiling LaTeX. Unknown error_msg)x = uy + v$.
The problem asks to find the remainder of$ (Error compiling LaTeX. Unknown error_msg)x + 2uyy2uyyx\boxed{\textbf{(D)}}$.
==Solution 3== If the statement is true for all values of$ (Error compiling LaTeX. Unknown error_msg)(x, y, u, v)(x, y, u, v)$.
If you let$ (Error compiling LaTeX. Unknown error_msg)x=43y = 8u = 5v = 3x + 2uy = 43 + 2 \cdot 5 \cdot 8 = 12383$.
When you plug in$ (Error compiling LaTeX. Unknown error_msg)u=5v = 30, 5, 10, 3, 6\boxed{\textbf{(D)}}$.
See Also
1964 AHSC (Problems • Answer Key • Resources) | ||
Preceded by Problem 2 |
Followed by Problem 4 | |
1 • 2 • 3 • 4 • 5 • 6 • 7 • 8 • 9 • 10 • 11 • 12 • 13 • 14 • 15 • 16 • 17 • 18 • 19 • 20 • 21 • 22 • 23 • 24 • 25 • 26 • 27 • 28 • 29 • 30 • 31 • 32 • 33 • 34 • 35 • 36 • 37 • 38 • 39 • 40 | ||
All AHSME Problems and Solutions |
The problems on this page are copyrighted by the Mathematical Association of America's American Mathematics Competitions.